You are on page 1of 23

AP PHYSICS 1 Test Booklet

Torque

1. This question is a short free-response question. Show your work for each part of the question. (10 points,
suggested time 15 minutes)

A uniform bar, of mass , with seven evenly spaced holes is held by sliding the bar over a horizontal peg
through one of the seven holes. The peg passes through hole C, and a cylinder hangs from a hook placed through
hole B as shown above. The mass of the bar is equal to the mass of the cylinder, and the location of the center of
mass of the bar is at the center of hole D. In this configuration, the bar-cylinder system remains motionless but is
free to rotate around the peg in hole C. Frictional forces acting on the bar are negligible.

(a) Two additional cylinders, identical to the one shown, can each be hung on the bar by hooks from any of the
open holes so that the bar will still be in rotational equilibrium when all three cylinders are hanging from it. From
which two holes should the two cylinders be hung? Briefly explain your reasoning.

The cylinder is moved to hang from hole A as shown in Case 1 above. The bar is released, and its resulting
angular acceleration is recorded. Then the cylinder is moved to hang from hole E, as shown in Case 2 above.
Again, the bar is released, and its resulting angular acceleration is recorded.

(b) Is the magnitude of the initial angular acceleration of the bar-cylinder system in Case 1 larger than, smaller
than, or equal to the magnitude of the initial angular acceleration of the bar-cylinder system in Case 2? In a clear,
coherent, paragraph-length response, explain your reasoning.

AP Physics 1 Page 1 of 23
Test Booklet

Torque

In a new scenario, a different uniform bar is held by a horizontal peg as shown above. The bar is initially
motionless but is free to rotate around the peg with negligible friction. A ball of clay is dropped onto the bar in the
position shown. The clay collides with and sticks to the bar.

(c) During this collision, is the linear momentum of the clay-bar system constant? Justify your answer.

(d) During this collision, is the angular momentum of the bar about an axis through the center of the bar constant?
Justify your answer.

Page 2 of 23 AP Physics 1
Test Booklet

Torque

2. This question is a short free-response question. Show your work for each part of the question. (10 points,
suggested time 15 minutes)

A uniform bar, of mass , with seven evenly spaced holes is held by sliding the bar over a horizontal peg
through one of the seven holes. The peg passes through hole D, and a cylinder hangs from a hook placed through
hole G as shown above. The mass of the bar is the same as the mass of the cylinder, and the location of the center
of mass of the bar is at the center of hole D. In this configuration, the bar-cylinder system is not in rotational
equilibrium and is free to rotate about the peg in hole D. Frictional forces acting on the bar are negligible.

(a) Two additional cylinders, identical to the one shown, can each be hung on the bar by hooks from any of the
open holes so that the bar will be in rotational equilibrium when all three cylinders are hanging from it. From
which two holes should the two cylinders be hung? Briefly explain your reasoning.

The bar is returned to its original setup, Case 1, in which the cylinder has weight , as shown above. The bar is
released, and its resulting angular acceleration is recorded. Then in Case 2, the weight is replaced by a string, as
shown above. A downward force of magnitude , equal to the weight of the hanging cylinder, is exerted on the
string. The resulting angular acceleration of the bar is recorded.

(b) Is the magnitude of the initial angular acceleration of the bar-cylinder system in Case 1 larger than, smaller

AP Physics 1 Page 3 of 23
Test Booklet

Torque

than, or equal to the magnitude of the initial angular acceleration of the bar in Case 2? In a clear, coherent,
paragraph-length response, explain your reasoning.

In a new scenario, a different uniform bar is held by a horizontal peg as shown above. The bar remains motionless
but is free to rotate around the peg with negligible friction. A lump of clay is dropped onto the bar in the position
shown. The clay collides with and sticks to the bar.

(c) During this collision, is the kinetic energy of the clay-bar system constant? Justify your answer.

(d) During this collision, is the angular momentum of the clay-bar system about an axis through the center of the
bar constant? Justify your answer.

Page 4 of 23 AP Physics 1
Test Booklet

Torque

3. This question is a short free-response question. Show your work for each part of the question.

( points, suggested time minutes)

The figures above show three stages of a dive performed by an athlete. During the dive, the athlete completes
several rotations in midair while traveling from the platform to the surface of the water. Figure shows the athlete
just after jumping off the platform. Figure shows the athlete rotating in midair. Figure shows the athlete about
to enter the water.

(a) In a clear, coherent paragraph-length response that may also contain figures and/or equations, explain why the
athlete’s angular speed increases between Figure and Figure but decreases between Figure and Figure .

(b) Is the rotational kinetic energy of the athlete in Figure greater than, less than, or equal to the
rotational kinetic energy of the athlete in Figure ?

Briefly explain your answer.

AP Physics 1 Page 5 of 23
Test Booklet

Torque

4. This question is a long free-response question. Show your work for each part of the question.

(12 points, suggested time 25 minutes)

A light string is attached to a block, passes over a pulley, and is wrapped around a T-shaped apparatus, as shown
above. The pulley has negligible rotational inertia and negligible friction, and the T-shaped apparatus rotates with
negligible friction. The system is released from rest. As the block accelerates downward, it pulls the string, which
causes the apparatus to start rotating.

Students in a physics lab first wish to determine the value of the block’s constant downward acceleration once it is
released from rest by observing it fall.

(a) Derive an equation for the acceleration of the block in terms of quantities that can be obtained from
measurements made using only a meterstick and a stopwatch.

(b) Design an experimental procedure the students could use to collect the data needed to determine the block’s
linear acceleration .

In the table at left below, list the quantities that would be measured in your experiment using the meterstick and
the stopwatch. Define a symbol to represent each quantity.

In the space to the right of and below the table, describe the overall procedure. Provide enough detail so that
another student could replicate the experiment, including any steps necessary to reduce experimental uncertainty.
As needed, use the symbols defined in the table.

If needed, you may include a simple diagram of the setup with your procedure.

Page 6 of 23 AP Physics 1
Test Booklet

Torque

Symbol
Quantity to Equipment for
for Procedure (and diagram, if needed)
be Measured Measurement
Quantity

Meterstick

Stopwatch

(c) Explain how the students should analyze the data from part (b) to determine the value of the block’s constant
linear acceleration .

(d) The students produce the data table shown below for five blocks with different masses.

AP Physics 1 Page 7 of 23
Test Booklet

Torque

Angular Torque
Acceleration Exerted on

Mass of Linear Acceleration of Tension in T-shaped


of T-shaped
Block Block Apparatus String Apparatus

by String

The students now want to determine the rotational inertia of the T-shaped apparatus.

i. Indicate below which quantities could be graphed to yield a straight line whose slope could be used to
calculate a numerical value for the rotational inertia of the T-shaped apparatus.

Vertical axis ________________________ Horizontal axis ________________________

ii. Use the grid below to plot the data points for the quantities indicated in part (d)(i). Clearly scale and label all
axes, including units as appropriate. Draw a straight line that best represents the data.

Page 8 of 23 AP Physics 1
Test Booklet

Torque

iii. Using the straight line from part (d)(ii), calculate a value for the rotational inertia of the T-shaped
apparatus.

AP Physics 1 Page 9 of 23
Test Booklet

Torque

5. This question is a long free-response question. Show your work for each part of the question. (15 points, suggested
time 25 minutes)

A vertical support rod is fixed at the center of a platform. A light string of length L is attached to the top of the
support rod, and the other end of the string is attached to a sphere of mass m, as shown above. The platform rotates
with an angular speed that can be varied. When the platform rotates with a constant angular speed (omega), the
string makes an angle (theta) with the rod.

(a) The platform rotates with a constant angular speed. Which of the above graphs correctly shows the magnitude
of the angular momentum of the platform as a function of time t about a vertical axis at the center of the support
rod? Briefly justify your answer.

(b) The platform rotates with a constant angular speed. Is the net torque exerted on the platform about its center
clockwise, counterclockwise, or zero? Explain your reasoning.

(c) As the period of rotation becomes very small, what value does approach? Without writing any equations or
formulas, justify your answer.

(d) A student derives an equation for the square of the period of revolution T of the sphere as a function of
. Indicate whether the equation is consistent with your answer in part (c) and explain
why.

(e) In a series of experimental trials, a group of students changes L and adjusts T to keep constant. They record L
and T for each trial. If the students use the equation given in part (d), what quantities could they graph to yield a
straight line whose slope could be used to find the acceleration due to gravity, g ?

Page 10 of 23 AP Physics 1
Test Booklet

Torque

(f) As the angular speed of the platform increases, does the period of revolution of the sphere increase, decrease,
or stay the same? Briefly explain your reasoning.

6.

A platform is initially rotating on smooth ice with negligible friction, as shown at left in the figure. A stationary disk
is dropped directly onto the center of the platform. A short time later, the disk and platform rotate together at the
same angular velocity, as shown at right in the figure. How does the angular momentum of only the platform
change, if at all, after the disk drops? What is a justification for your answer?
(A) It decreases. The top disk exerts a torque on the platform.
(B) It decreases. The potential energy of the platform-disk-Earth system decreases when the disk drops.
(C) It stays the same. Angular momentum is a conserved quantity.
It stays the same. The torques that the disk and platform exert on each other are equal in magnitude but in
(D)
opposite directions.

7.

The graph shows the angular momentum of a wheel about its axle as a function of time . What is the magnitude
of the net torque on the wheel?
(A)

(B)
(C)
(D)

AP Physics 1 Page 11 of 23
Test Booklet

Torque

8.

A horizontal disk is at rest on top of an axle, and the friction between the disk and axle is not negligible. In
experiment 1, an applied torque is exerted to the edge of the disk for . At that moment, the applied torque is
removed, and the disk eventually comes to rest as a result of a frictional torque. Graphs of the disk’s angular
momentum as a function of time are shown in Figures 1 and 2 for the two experiments. Which of the following
statements is correct about the applied torque and frictional torque exerted on the disk in
experiment 1 and experiment 2 ?

Page 12 of 23 AP Physics 1
Test Booklet

Torque

Applied Torque Frictional Torque


(A)

Applied Torque Frictional Torque


(B)

Applied Torque Frictional Torque


(C)

Applied Torque Frictional Torque


(D)

9.

The figure above shows a uniform meterstick that is set on a fulcrum at its center. A force of magnitude F toward
the bottom of the page is exerted on the meterstick at the position shown. At which of the labeled positions must an
upward force of magnitude 2F be exerted on the meterstick to keep the meterstick in equilibrium?
(A) A
(B) B
(C) C
(D) D

AP Physics 1 Page 13 of 23
Test Booklet

Torque

10.

A bucket is attached to a long, lightweight string that is wrapped around a pulley that has nonnegligible mass, as
shown. The pulley is given an initial clockwise rotation and then rotates with negligible friction. The angular
momentum of the pulley about the pulley’s center has magnitude , and the torque exerted by the string on the
pulley about the pulley’s center has magnitude . Which of the following correctly describes the changes, if any, in
and while the pulley is rotating clockwise?

A Stays constant Stays constant


B Stays constant Decreases
C Decreases Stays constant
D Decreases Decreases

(A) A
(B) B
(C) C
(D) D

11. The wheel on a vehicle has a rotational inertia of 2.0 kg⋅m2. At the instant the wheel has a counterclockwise angular
velocity of 6.0 rad/s, an average counterclockwise torque of 5.0 N⋅m is applied, and continues for 4.0 s. What is the
change in angular momentum of the wheel?
(A) 12 kg⋅m2/s
(B) 16 kg⋅m2/s
(C) 20 kg⋅m2/s
(D) 32 kg⋅m2/s

Page 14 of 23 AP Physics 1
Test Booklet

Torque

A disk of mass slides with negligible friction to the right with speed on a horizontal table. The disk collides
elastically with a uniform rod of length that is at rest and free to pivot about one end, as shown in Figure 1 above. The
disk rebounds to the left with speed , and the rod rotates with friction and stops at a final angle from its initial
position, as shown in Figure 2.

12. What happens to the magnitude of the angular momentum of the disk-rod system with respect to the pivot during
the time interval between Figure 1 and Figure 2, and why?
(A) It decreases because the disk reverses its direction of motion.
(B) It decreases because an external torque is exerted by friction.
(C) It does not change because there was no angular momentum before the collision.
(D) It does not change because angular momentum was conserved during the collision.

AP Physics 1 Page 15 of 23
Test Booklet

Torque

A solid disk whose plane is parallel to the ground spins with an initial angular speed . Three identical blocks are
dropped onto the disk at locations , , and , one at a time, not necessarily in that order. Each block instantaneously
sticks to the surface of the disk, slowing the disk’s rotation. A graph of the angular speed of the disk as a function of time
is shown.

13. Based on the data presented in the graph, which of the following lists the points in the order in which the blocks are
dropped onto the disk?
(A)
(B)
(C)
(D)

Page 16 of 23 AP Physics 1
Test Booklet

Torque

14.

A disk is initially rotating counterclockwise around a fixed axis with angular speed w0.

At time t = 0, the two forces shown in the figure above are exerted on the disk. If counterclockwise is positive,
which of the following could show the angular velocity of the disk as a function of time?

(A)

(B)

(C)

(D)

AP Physics 1 Page 17 of 23
Test Booklet

Torque

15.

The figure above represents a stick of uniform density that is attached to a pivot at the right end and has equally
spaced marks along its length. Any one of the four forces shown can be exerted on the stick as indicated. Which
force will create the largest rate of change in the stick’s angular momentum?
(A) The 30 N force
(B) The 40 N force
(C) The 60 N force
(D) The 150 N force

16. A horizontal disk of radius rotates about its center with an angular velocity of . The edge of
the horizontal disk is placed in contact with a wall, and the disk comes to rest after . Which of the following
situations associated with linear impulse is analogous to the angular impulse that is described?
A block is initially at rest. An applied force of is applied to the block, but the block does not
(A)
move.
(B) A block is initially at rest. A net force of is applied to the block until it has a speed of .
A block is initially traveling at . An applied force of is applied to the block in the
(C)
direction of its velocity vector for .
A block is initially traveling at . The block encounters a frictional force until the block
(D)
eventually stops.

Page 18 of 23 AP Physics 1
Test Booklet

Torque

17.

A thin rod of length d on a frictionless surface is pivoted about one end, as shown above, and can rotate freely. The
rod is at rest when it is struck by a sphere with linear momentum of magnitude pi perpendicular to the rod. The
sphere rebounds along its original line of motion with momentum of magnitude pf. What is the magnitude of the
angular momentum of the rod immediately after the collision?
(A) pf - pi
(B) pf + pi
(C) (pf - pi )d
(D) (pf + pi )d

18.

Steel sphere A of mass M is moving along a horizontal surface with constant speed v. Identical steel sphere B is at
rest and hangs on a string of length R attached to a support at point P, as shown in the figure above. The spheres
collide, and as a result sphere A stops and sphere B swings a vertical height h before coming momentarily to rest.
Knowing values for which of the following will allow determination of the angular impulse on sphere B with
respect to P due to the collision?
(A) M and v only
(B) M, v, and h
(C) R and h
(D) R, M, and v

AP Physics 1 Page 19 of 23
Test Booklet

Torque

19.

In an experiment, students must determine the angular momentum of a vertically rotating nonuniform disk of
diameter . A central axle of negligible friction is located at the center of the disk and is oriented
perpendicular to the plane of the page. A string is wound around the disk, and an object of unknown mass is
attached to the string, as shown in Figure 1. The object is released from rest, which causes the disk to rotate as the
object falls. The disk’s angular position as a function of time is shown in Figure 2. The tension in the string is
and remains constant. Which of the following mathematical routines could a student use to determine the change in
angular momentum of the disk during a time interval ? Justify your selection.
(A) Use , where with , , and .
(B) Use , where with , , and .
(C) Use , where with , , and .
(D) Use , where with , , and .

20.

In an experiment, an external torque is applied to the edge of a disk of radius such that the edge of the disk
speeds up as it continues to rotate. The tangential speed as a function of time is shown for the edge of the disk. The
rotational inertia of the disk is . Can a student use the graph and the known information to calculate
the net torque exerted on the edge of the disk?

Page 20 of 23 AP Physics 1
Test Booklet

Torque

Yes, because the slope of the graph multiplied by the rotational inertia is equal to the net torque exerted on
(A)
the edge of the disk.
Yes, because the change in tangential speed per unit of time can be multiplied by the rotational inertia
(B)
divided by the radius of the disk.
No, because the net force exerted on the edge of the disk must first be determined before the net torque can
(C)
be calculated.
No, because the change in tangential speed of the edge of the disk per unit of time cannot be used to
(D)
determine the angular acceleration of the disk.

21.

Forces are exerted on identical rods , , and . The rods are each free to rotate about the pivot on the left end
of the rod. The forces exerted on rods and are applied to the midpoints of the rods. Which of the following
identifies the rod that has the same torque about as Rod and provides correct reasoning?
Rod , because the magnitude of the force exerted on Rod is equal to the magnitude of the force exerted
(A)
on Rod .
Rod , because the magnitude of the perpendicular component of the force exerted on Rod is half the
(B)
magnitude of the force exerted on Rod , but the force on Rod is exerted at twice the distance from .
Rod , because the magnitude of the perpendicular component of the force exerted on Rod is equal to the
(C)
magnitude of the force exerted on Rod .
Rod , because the magnitude of the force exerted on Rod is twice the magnitude of the force exerted on
(D)
Rod , but the force on Rod is exerted at twice the distance from .

AP Physics 1 Page 21 of 23
Test Booklet

Torque

22.

Two objects, X and Y, experience external net torques that vary over a period of seconds. Object X has a moment
of inertia , and Object Y has a moment of inertia . The average value of the magnitude of the external net
torque exerted on Object X from time to is . Similarly, the average value for Object Y is . The
magnitudes of the angular momenta of Objects X and Y versus are shown in the graph. Which of the following
expressions correctly relates to ?
(A)
(B)
(C)
(D)

23.

Two identical disks rotate about their centers in opposite directions with the same magnitude of angular speed .
The top disk is dropped onto the bottom disk, as shown in the figure, so they collide and stick together. Which of
the following predictions is correct about the motion of each individual disk after the collision?
(A) Each disk will spin with the same final angular velocity where .
(B) Each disk will spin with the same final angular velocity where .
(C) Each disk will spin with the same final angular velocity where .
(D) Each disk will spin with the same final angular velocity where .

Page 22 of 23 AP Physics 1
Test Booklet

Torque

24.

A uniform meterstick is balanced at the center, as shown above. Which of the following shows how a 0.50 kg mass
and a 1.0 kg mass could be hung on the meterstick so that the stick stays balanced?

(A)

(B)

(C)

(D)

(E)

AP Physics 1 Page 23 of 23

You might also like